Any other quick way to find the remainder? [closed]












1












$begingroup$


I want to find
$$19^{13}operatorname{mod}(2537).$$



Is there quick way to find the remainder?










share|cite|improve this question











$endgroup$



closed as off-topic by Saad, Davide Giraudo, user10354138, Namaste, Paul Frost Dec 15 '18 at 23:11


This question appears to be off-topic. The users who voted to close gave this specific reason:


  • "This question is missing context or other details: Please provide additional context, which ideally explains why the question is relevant to you and our community. Some forms of context include: background and motivation, relevant definitions, source, possible strategies, your current progress, why the question is interesting or important, etc." – Saad, Davide Giraudo, user10354138, Namaste, Paul Frost

If this question can be reworded to fit the rules in the help center, please edit the question.












  • 4




    $begingroup$
    $$(19,43)=1=(19,59), 2537=43cdot59$$
    $endgroup$
    – lab bhattacharjee
    Dec 15 '18 at 9:56






  • 2




    $begingroup$
    Expounding on what Peter said, since $19$ is prime, only $1, 19, 19^2, 19^3, ..., 19^{13}$ divide $19^{13}$, and $19$ and any power of it does not divide $2537$.
    $endgroup$
    – Eevee Trainer
    Dec 15 '18 at 9:57










  • $begingroup$
    @ Peter @ Eevee Trainer @ lab bhattacharjee Yes, my question is wrong because my calculation is incorrect. Now I know the answer. Thanks.
    $endgroup$
    – Ongky Denny Wijaya
    Dec 15 '18 at 10:47
















1












$begingroup$


I want to find
$$19^{13}operatorname{mod}(2537).$$



Is there quick way to find the remainder?










share|cite|improve this question











$endgroup$



closed as off-topic by Saad, Davide Giraudo, user10354138, Namaste, Paul Frost Dec 15 '18 at 23:11


This question appears to be off-topic. The users who voted to close gave this specific reason:


  • "This question is missing context or other details: Please provide additional context, which ideally explains why the question is relevant to you and our community. Some forms of context include: background and motivation, relevant definitions, source, possible strategies, your current progress, why the question is interesting or important, etc." – Saad, Davide Giraudo, user10354138, Namaste, Paul Frost

If this question can be reworded to fit the rules in the help center, please edit the question.












  • 4




    $begingroup$
    $$(19,43)=1=(19,59), 2537=43cdot59$$
    $endgroup$
    – lab bhattacharjee
    Dec 15 '18 at 9:56






  • 2




    $begingroup$
    Expounding on what Peter said, since $19$ is prime, only $1, 19, 19^2, 19^3, ..., 19^{13}$ divide $19^{13}$, and $19$ and any power of it does not divide $2537$.
    $endgroup$
    – Eevee Trainer
    Dec 15 '18 at 9:57










  • $begingroup$
    @ Peter @ Eevee Trainer @ lab bhattacharjee Yes, my question is wrong because my calculation is incorrect. Now I know the answer. Thanks.
    $endgroup$
    – Ongky Denny Wijaya
    Dec 15 '18 at 10:47














1












1








1





$begingroup$


I want to find
$$19^{13}operatorname{mod}(2537).$$



Is there quick way to find the remainder?










share|cite|improve this question











$endgroup$




I want to find
$$19^{13}operatorname{mod}(2537).$$



Is there quick way to find the remainder?







elementary-number-theory






share|cite|improve this question















share|cite|improve this question













share|cite|improve this question




share|cite|improve this question








edited Dec 15 '18 at 22:22









Namaste

1




1










asked Dec 15 '18 at 9:54









Ongky Denny WijayaOngky Denny Wijaya

3748




3748




closed as off-topic by Saad, Davide Giraudo, user10354138, Namaste, Paul Frost Dec 15 '18 at 23:11


This question appears to be off-topic. The users who voted to close gave this specific reason:


  • "This question is missing context or other details: Please provide additional context, which ideally explains why the question is relevant to you and our community. Some forms of context include: background and motivation, relevant definitions, source, possible strategies, your current progress, why the question is interesting or important, etc." – Saad, Davide Giraudo, user10354138, Namaste, Paul Frost

If this question can be reworded to fit the rules in the help center, please edit the question.







closed as off-topic by Saad, Davide Giraudo, user10354138, Namaste, Paul Frost Dec 15 '18 at 23:11


This question appears to be off-topic. The users who voted to close gave this specific reason:


  • "This question is missing context or other details: Please provide additional context, which ideally explains why the question is relevant to you and our community. Some forms of context include: background and motivation, relevant definitions, source, possible strategies, your current progress, why the question is interesting or important, etc." – Saad, Davide Giraudo, user10354138, Namaste, Paul Frost

If this question can be reworded to fit the rules in the help center, please edit the question.








  • 4




    $begingroup$
    $$(19,43)=1=(19,59), 2537=43cdot59$$
    $endgroup$
    – lab bhattacharjee
    Dec 15 '18 at 9:56






  • 2




    $begingroup$
    Expounding on what Peter said, since $19$ is prime, only $1, 19, 19^2, 19^3, ..., 19^{13}$ divide $19^{13}$, and $19$ and any power of it does not divide $2537$.
    $endgroup$
    – Eevee Trainer
    Dec 15 '18 at 9:57










  • $begingroup$
    @ Peter @ Eevee Trainer @ lab bhattacharjee Yes, my question is wrong because my calculation is incorrect. Now I know the answer. Thanks.
    $endgroup$
    – Ongky Denny Wijaya
    Dec 15 '18 at 10:47














  • 4




    $begingroup$
    $$(19,43)=1=(19,59), 2537=43cdot59$$
    $endgroup$
    – lab bhattacharjee
    Dec 15 '18 at 9:56






  • 2




    $begingroup$
    Expounding on what Peter said, since $19$ is prime, only $1, 19, 19^2, 19^3, ..., 19^{13}$ divide $19^{13}$, and $19$ and any power of it does not divide $2537$.
    $endgroup$
    – Eevee Trainer
    Dec 15 '18 at 9:57










  • $begingroup$
    @ Peter @ Eevee Trainer @ lab bhattacharjee Yes, my question is wrong because my calculation is incorrect. Now I know the answer. Thanks.
    $endgroup$
    – Ongky Denny Wijaya
    Dec 15 '18 at 10:47








4




4




$begingroup$
$$(19,43)=1=(19,59), 2537=43cdot59$$
$endgroup$
– lab bhattacharjee
Dec 15 '18 at 9:56




$begingroup$
$$(19,43)=1=(19,59), 2537=43cdot59$$
$endgroup$
– lab bhattacharjee
Dec 15 '18 at 9:56




2




2




$begingroup$
Expounding on what Peter said, since $19$ is prime, only $1, 19, 19^2, 19^3, ..., 19^{13}$ divide $19^{13}$, and $19$ and any power of it does not divide $2537$.
$endgroup$
– Eevee Trainer
Dec 15 '18 at 9:57




$begingroup$
Expounding on what Peter said, since $19$ is prime, only $1, 19, 19^2, 19^3, ..., 19^{13}$ divide $19^{13}$, and $19$ and any power of it does not divide $2537$.
$endgroup$
– Eevee Trainer
Dec 15 '18 at 9:57












$begingroup$
@ Peter @ Eevee Trainer @ lab bhattacharjee Yes, my question is wrong because my calculation is incorrect. Now I know the answer. Thanks.
$endgroup$
– Ongky Denny Wijaya
Dec 15 '18 at 10:47




$begingroup$
@ Peter @ Eevee Trainer @ lab bhattacharjee Yes, my question is wrong because my calculation is incorrect. Now I know the answer. Thanks.
$endgroup$
– Ongky Denny Wijaya
Dec 15 '18 at 10:47










2 Answers
2






active

oldest

votes


















2












$begingroup$

HINT:




  • Write $19^{13}=(19^3)^4cdot19$.


  • Note that $19^3=6859equiv-752pmod{2537}$.


  • Write $(-752)^4$ as $((-752)^2)^2$ and evaluate the inner bracket to simplify.







share|cite|improve this answer









$endgroup$













  • $begingroup$
    Please check my answer, is it correct answer?
    $endgroup$
    – Ongky Denny Wijaya
    Dec 15 '18 at 10:43










  • $begingroup$
    Quite rather a PSQ, don't you think?
    $endgroup$
    – Namaste
    Dec 15 '18 at 22:23










  • $begingroup$
    @amWhy They originally had some working but it was pointed out that it was wrong so they deleted it. However, they have provided their own correct answer below mine so I won't be too harsh about it.
    $endgroup$
    – TheSimpliFire
    Dec 16 '18 at 8:56





















2












$begingroup$

begin{eqnarray}
19^3=6859&equiv& 1785(operatorname{mod}2537)\
19^6=left(19^3right)^2equiv (1785)^2=3186225&equiv& 2290(operatorname{mod}2537)\
19^{12}=left(19^6right)^2equiv (2290)^2 = 5244100&equiv& 121(operatorname{mod}2537)\
19^{13}=19^{12}cdot19&equiv& 121cdot 19=2299(operatorname{mod}2537)
end{eqnarray}



So the remainder is 2299. Is this correct answer?






share|cite|improve this answer









$endgroup$









  • 1




    $begingroup$
    Yes${}{}{}{}{}{}$
    $endgroup$
    – TheSimpliFire
    Dec 15 '18 at 10:55










  • $begingroup$
    Thank you very much for your hint, @ TheSimpliFire.
    $endgroup$
    – Ongky Denny Wijaya
    Dec 15 '18 at 11:00


















2 Answers
2






active

oldest

votes








2 Answers
2






active

oldest

votes









active

oldest

votes






active

oldest

votes









2












$begingroup$

HINT:




  • Write $19^{13}=(19^3)^4cdot19$.


  • Note that $19^3=6859equiv-752pmod{2537}$.


  • Write $(-752)^4$ as $((-752)^2)^2$ and evaluate the inner bracket to simplify.







share|cite|improve this answer









$endgroup$













  • $begingroup$
    Please check my answer, is it correct answer?
    $endgroup$
    – Ongky Denny Wijaya
    Dec 15 '18 at 10:43










  • $begingroup$
    Quite rather a PSQ, don't you think?
    $endgroup$
    – Namaste
    Dec 15 '18 at 22:23










  • $begingroup$
    @amWhy They originally had some working but it was pointed out that it was wrong so they deleted it. However, they have provided their own correct answer below mine so I won't be too harsh about it.
    $endgroup$
    – TheSimpliFire
    Dec 16 '18 at 8:56


















2












$begingroup$

HINT:




  • Write $19^{13}=(19^3)^4cdot19$.


  • Note that $19^3=6859equiv-752pmod{2537}$.


  • Write $(-752)^4$ as $((-752)^2)^2$ and evaluate the inner bracket to simplify.







share|cite|improve this answer









$endgroup$













  • $begingroup$
    Please check my answer, is it correct answer?
    $endgroup$
    – Ongky Denny Wijaya
    Dec 15 '18 at 10:43










  • $begingroup$
    Quite rather a PSQ, don't you think?
    $endgroup$
    – Namaste
    Dec 15 '18 at 22:23










  • $begingroup$
    @amWhy They originally had some working but it was pointed out that it was wrong so they deleted it. However, they have provided their own correct answer below mine so I won't be too harsh about it.
    $endgroup$
    – TheSimpliFire
    Dec 16 '18 at 8:56
















2












2








2





$begingroup$

HINT:




  • Write $19^{13}=(19^3)^4cdot19$.


  • Note that $19^3=6859equiv-752pmod{2537}$.


  • Write $(-752)^4$ as $((-752)^2)^2$ and evaluate the inner bracket to simplify.







share|cite|improve this answer









$endgroup$



HINT:




  • Write $19^{13}=(19^3)^4cdot19$.


  • Note that $19^3=6859equiv-752pmod{2537}$.


  • Write $(-752)^4$ as $((-752)^2)^2$ and evaluate the inner bracket to simplify.








share|cite|improve this answer












share|cite|improve this answer



share|cite|improve this answer










answered Dec 15 '18 at 10:06









TheSimpliFireTheSimpliFire

12.7k62461




12.7k62461












  • $begingroup$
    Please check my answer, is it correct answer?
    $endgroup$
    – Ongky Denny Wijaya
    Dec 15 '18 at 10:43










  • $begingroup$
    Quite rather a PSQ, don't you think?
    $endgroup$
    – Namaste
    Dec 15 '18 at 22:23










  • $begingroup$
    @amWhy They originally had some working but it was pointed out that it was wrong so they deleted it. However, they have provided their own correct answer below mine so I won't be too harsh about it.
    $endgroup$
    – TheSimpliFire
    Dec 16 '18 at 8:56




















  • $begingroup$
    Please check my answer, is it correct answer?
    $endgroup$
    – Ongky Denny Wijaya
    Dec 15 '18 at 10:43










  • $begingroup$
    Quite rather a PSQ, don't you think?
    $endgroup$
    – Namaste
    Dec 15 '18 at 22:23










  • $begingroup$
    @amWhy They originally had some working but it was pointed out that it was wrong so they deleted it. However, they have provided their own correct answer below mine so I won't be too harsh about it.
    $endgroup$
    – TheSimpliFire
    Dec 16 '18 at 8:56


















$begingroup$
Please check my answer, is it correct answer?
$endgroup$
– Ongky Denny Wijaya
Dec 15 '18 at 10:43




$begingroup$
Please check my answer, is it correct answer?
$endgroup$
– Ongky Denny Wijaya
Dec 15 '18 at 10:43












$begingroup$
Quite rather a PSQ, don't you think?
$endgroup$
– Namaste
Dec 15 '18 at 22:23




$begingroup$
Quite rather a PSQ, don't you think?
$endgroup$
– Namaste
Dec 15 '18 at 22:23












$begingroup$
@amWhy They originally had some working but it was pointed out that it was wrong so they deleted it. However, they have provided their own correct answer below mine so I won't be too harsh about it.
$endgroup$
– TheSimpliFire
Dec 16 '18 at 8:56






$begingroup$
@amWhy They originally had some working but it was pointed out that it was wrong so they deleted it. However, they have provided their own correct answer below mine so I won't be too harsh about it.
$endgroup$
– TheSimpliFire
Dec 16 '18 at 8:56













2












$begingroup$

begin{eqnarray}
19^3=6859&equiv& 1785(operatorname{mod}2537)\
19^6=left(19^3right)^2equiv (1785)^2=3186225&equiv& 2290(operatorname{mod}2537)\
19^{12}=left(19^6right)^2equiv (2290)^2 = 5244100&equiv& 121(operatorname{mod}2537)\
19^{13}=19^{12}cdot19&equiv& 121cdot 19=2299(operatorname{mod}2537)
end{eqnarray}



So the remainder is 2299. Is this correct answer?






share|cite|improve this answer









$endgroup$









  • 1




    $begingroup$
    Yes${}{}{}{}{}{}$
    $endgroup$
    – TheSimpliFire
    Dec 15 '18 at 10:55










  • $begingroup$
    Thank you very much for your hint, @ TheSimpliFire.
    $endgroup$
    – Ongky Denny Wijaya
    Dec 15 '18 at 11:00
















2












$begingroup$

begin{eqnarray}
19^3=6859&equiv& 1785(operatorname{mod}2537)\
19^6=left(19^3right)^2equiv (1785)^2=3186225&equiv& 2290(operatorname{mod}2537)\
19^{12}=left(19^6right)^2equiv (2290)^2 = 5244100&equiv& 121(operatorname{mod}2537)\
19^{13}=19^{12}cdot19&equiv& 121cdot 19=2299(operatorname{mod}2537)
end{eqnarray}



So the remainder is 2299. Is this correct answer?






share|cite|improve this answer









$endgroup$









  • 1




    $begingroup$
    Yes${}{}{}{}{}{}$
    $endgroup$
    – TheSimpliFire
    Dec 15 '18 at 10:55










  • $begingroup$
    Thank you very much for your hint, @ TheSimpliFire.
    $endgroup$
    – Ongky Denny Wijaya
    Dec 15 '18 at 11:00














2












2








2





$begingroup$

begin{eqnarray}
19^3=6859&equiv& 1785(operatorname{mod}2537)\
19^6=left(19^3right)^2equiv (1785)^2=3186225&equiv& 2290(operatorname{mod}2537)\
19^{12}=left(19^6right)^2equiv (2290)^2 = 5244100&equiv& 121(operatorname{mod}2537)\
19^{13}=19^{12}cdot19&equiv& 121cdot 19=2299(operatorname{mod}2537)
end{eqnarray}



So the remainder is 2299. Is this correct answer?






share|cite|improve this answer









$endgroup$



begin{eqnarray}
19^3=6859&equiv& 1785(operatorname{mod}2537)\
19^6=left(19^3right)^2equiv (1785)^2=3186225&equiv& 2290(operatorname{mod}2537)\
19^{12}=left(19^6right)^2equiv (2290)^2 = 5244100&equiv& 121(operatorname{mod}2537)\
19^{13}=19^{12}cdot19&equiv& 121cdot 19=2299(operatorname{mod}2537)
end{eqnarray}



So the remainder is 2299. Is this correct answer?







share|cite|improve this answer












share|cite|improve this answer



share|cite|improve this answer










answered Dec 15 '18 at 10:41









Ongky Denny WijayaOngky Denny Wijaya

3748




3748








  • 1




    $begingroup$
    Yes${}{}{}{}{}{}$
    $endgroup$
    – TheSimpliFire
    Dec 15 '18 at 10:55










  • $begingroup$
    Thank you very much for your hint, @ TheSimpliFire.
    $endgroup$
    – Ongky Denny Wijaya
    Dec 15 '18 at 11:00














  • 1




    $begingroup$
    Yes${}{}{}{}{}{}$
    $endgroup$
    – TheSimpliFire
    Dec 15 '18 at 10:55










  • $begingroup$
    Thank you very much for your hint, @ TheSimpliFire.
    $endgroup$
    – Ongky Denny Wijaya
    Dec 15 '18 at 11:00








1




1




$begingroup$
Yes${}{}{}{}{}{}$
$endgroup$
– TheSimpliFire
Dec 15 '18 at 10:55




$begingroup$
Yes${}{}{}{}{}{}$
$endgroup$
– TheSimpliFire
Dec 15 '18 at 10:55












$begingroup$
Thank you very much for your hint, @ TheSimpliFire.
$endgroup$
– Ongky Denny Wijaya
Dec 15 '18 at 11:00




$begingroup$
Thank you very much for your hint, @ TheSimpliFire.
$endgroup$
– Ongky Denny Wijaya
Dec 15 '18 at 11:00



Popular posts from this blog

Plaza Victoria

In PowerPoint, is there a keyboard shortcut for bulleted / numbered list?

How to put 3 figures in Latex with 2 figures side by side and 1 below these side by side images but in...